Skip to content
New issue

Have a question about this project? Sign up for a free GitHub account to open an issue and contact its maintainers and the community.

By clicking “Sign up for GitHub”, you agree to our terms of service and privacy statement. We’ll occasionally send you account related emails.

Already on GitHub? Sign in to your account

added exercice 5.1 #6

Open
wants to merge 1 commit into
base: master
Choose a base branch
from
Open
Changes from all commits
Commits
File filter

Filter by extension

Filter by extension

Conversations
Failed to load comments.
Loading
Jump to
Jump to file
Failed to load files.
Loading
Diff view
Diff view
38 changes: 38 additions & 0 deletions chapter/chapter5.tex
Original file line number Diff line number Diff line change
@@ -0,0 +1,38 @@
\chapter{The quantum Fourier transform and it's application}


\Textbf{5.1}


We can write the Fourier quantum transformation as a matrix U.
By definition, for an arbitrary ket $|x\rangle = \sum_{j=0}^{N-1} x_j | j \rangle$, the
quantum fourier transform maps $|x\rangle$ to $|y\rangle$ as such, noting $w = e^{2 i \pi /N}$ :

$$F(|x\rangle) = |y\rangle = \frac{1}{\sqrt{N}} \sum_{j=0}^{N-1} \sum_{n=0}^{N-1} x_n w^{k n} | j \rangle$$

the Fourier quantum transformation can then be written as a matrix equation, giving the Fourier transform's
matrix, $F |x\rangle = |y\rangle$, with :

$$ F = \frac{1}{\sqrt{N}} \begin{bmatrix}
1 & 1 & \hdots & 1 \\
1 & w & \hdots & w^{N-1} \\
1 & w^2 & \hdots & w^{2(N-1)} \\
\vdots & \vdots & \vdots & \vdots \\
1 & w^{N-1} & \hdots & w^{(N-1)(N-1)}
\end{bmatrix}.$$

Let's now show that this matrix is indeed unitary.
If we note $A = F F^{\dagger}$, then
$$A_{j,k} = \sum_{n=1}^N F_{j,n} F^{\dagger}_{n,k} = \frac{1}{N} (1 + w^{j-n}+w^{2(j-n)}+...+w^{(N-1)(j-n)})$$.

If $j=n$ then $A_{j,k} = \frac{N}{N} = 1$.

If $j \neq n$ then $A_{j,k} = 0$ according to the following orthogonal property over the roots of unity :

$$ \sum_{k=0}^{N-1} (z^{jk})^\dagger z^{j' k} = 0 \quad \mathtt{if } \quad j \neq j'$$

We then see that $F F^{\dagger} = I$.
It is straightforward to show that $F F^{\dagger} =F^{\dagger} F$, we can then conclude that
the quantum Fourier transform is unitary.